Was macht eine Symmetrie, die den Lagrange-Operator durch eine totale Ableitung ändert, mit dem Hamilton-Operator HHH?

Eine winzige Symmetrietransformation kann die Lagrange-Funktion verändern L durch eine Gesamtzeitableitung einer Funktion F . Dies ist eine grundlegende Tatsache, die im Beweis des Satzes von Noether verwendet wird.

Wie können wir den Effekt dieses totalen Ableitungsterms im Hamilton-System sehen? Gibt es ein gutes Beispiel zum Ausarbeiten? Mir fällt auf Anhieb keiner ein. Es erscheint mir nur seltsam, dass all diese Aufregung um totale Ableitungen im Hamiltonschen Rahmen zu verschwinden scheint.

Antworten (3)

i) Haftungsausschluss. Als Purist missbillige ich die gängige Praxis, die Implikation zu nennen

(1) { Q , H } + Q T   =   0 D Q D T     0.
für eine „Hamiltonsche Version des Satzes von Noether“ vgl. meine Phys.SE antwortet hier & hier . Mein Grund ist, dass die Implikation (1) nur eine triviale Konsequenz der Hamiltonschen Gleichungen ist, nicht mehr.

II) Stattdessen sollte sich eine „Hamiltonsche Version des Satzes von Noether“ auf Quasi-Symmetrien einer Hamiltonschen Wirkung beziehen

(2) S H [ Q , P ]   :=   D T   L H ( Q , Q ˙ , P , T ) ,
und ihre entsprechenden Erhaltungsgesetze. Hier L H ist die sogenannte Hamiltonsche Lagrangedichte
(3) L H ( Q , Q ˙ , P , T )   :=   ich = 1 N P ich Q ˙ ich H ( Q , P , T ) .

III) Es ist ein Missverständnis, dass all diese Aufregung um totale Ableitungen [...] im Hamiltonschen Rahmen verschwindet. Die hamiltonsche Version erlaubt, dass die hamiltonsche Wirkung nur bis zu den Randtermen (dh einer sogenannten Quasi-Symmetrie ) invariant ist, genau wie in der Lagrange-Standardformulierung des Satzes von Noether . Siehe auch diesen verwandten Phys.SE-Beitrag.

Ich nehme an, meine Frage könnte dann verfeinert werden: Gibt es im Hamiltonschen Rahmen einen grundlegenden Unterschied zwischen einer Symmetrie für die δ L = 0 und eine für die δ L = F ˙ ?
Sie benötigen nur die letzte Bedingung für den Satz von Noether.

Ich nehme an, ich habe die "Antwort" auf meine sehr vage Frage herausgefunden, obwohl die anderen Antworten hier auch hilfreich sind. Der "Hamiltonian Lagrangian" ist

L = P ich Q ˙ ich H .
Angenommen, wir haben eine Erhaltungsladung Q , das ist
{ Q , H } = 0.
Wenn wir die winzige Symmetrievariation machen
δ Q ich = Q P ich δ P ich = Q Q ich
Dann
δ L = Q Q ich Q ˙ ich P ich D D T ( Q P ich ) + { H , Q } = Q P ich Q ˙ ich P ˙ ich Q P ich + D D T ( P ich Q P ich ) = D D T ( P ich Q P ich Q )

Das können wir also sehen L zwangsläufig durch eine totale Ableitung ändert. Wenn die Menge P ich Q P ich Q = 0 , die totale Ableitung ist 0 . Dies geschieht, wenn die Erhaltungsgröße von der Form ist

Q = P ich F ich ( Q ) .
Beachten Sie, dass im obigen Fall
δ Q ich = F ich ( Q )
Das heißt, Symmetrietransformationen, die die nicht "verwechseln". P ist mit dem Q haben keinen Gesamtableitungsterm in δ L .

Der Grund, warum wir nicht von "Ändern des Hamilton-Operators durch eine totale Ableitung" sprechen, liegt darin, dass Symmetrien und Erhaltungssätze im Hamilton-Bild normalerweise anders gehandhabt werden.

In der Hamiltonschen Mechanik jede Funktion F auf dem Phasenraum erzeugt einen Fluss auf dem Phasenraum, dh eine Ein-Parameter-Familie von kanonischen Transformationen ( Q , P ) ( Q ~ ( a ) , P ~ ( a ) ) . Die induzierte Änderungsrate einer beliebigen anderen Phasenraumfunktion G Ist

D G D a = { G , F } .
Insbesondere erzeugt der Hamiltonian selbst eine Zeitübersetzung,
D G D T = { G , H } .
Die Aussage, dass Q ( Q , P ) ist eine Erhaltungsgröße ist einfach
{ Q , H } = 0.
Das heißt, die Zeitentwicklung, die durch erzeugt wird H ändert den Wert von nicht Q . Der Schlüssel ist, dass dies durch die Antisymmetrie der Poisson-Klammer äquivalent ist zu { H , Q } = 0 , die besagt, dass die kanonischen Transformationen von erzeugt werden Q ändern Sie nicht die Werte von H .

Somit gilt bei gegebener infinitesimaler kanonischer Transformation H ebenso ist sein Erzeuger eine Erhaltungsgröße. Dies kommt dem Satz von Noether am nächsten, den Sie normalerweise in der Hamiltonschen Mechanik sehen werden. Da bezieht es sich nur auf H , nicht das Integral von H , über Aufbewahrung braucht man nicht zu reden H invariant bis zu einer totalen Ableitung – es muss nur invariant sein, Punkt. (Siehe aber auch die Antwort von Qmechanic über eine aktionsähnliche Formulierung, wo sie erscheint.)